Answer choice E
Would you explain why answer choice E is wrong? Assume: 1, O G R 2, O P R 3, Y P R
Faezeh on January 15 at 03:40PM
  • December 2010 LSAT
  • SEC4
  • Q11
1
Reply
P or O in every group
Hello I was watching one of the question explanations and one of the deductions made was that O o...
MadisonVanNatter on January 15 at 03:39PM
  • December 2010 LSAT
  • SEC4
  • Q7
1
Reply
Question 6
The question wording threw me off. I watched the video and it helped clarify a bit, but I am stil...
nicolebet on January 15 at 03:35PM
  • December 2010 LSAT
  • SEC4
  • Q6
1
Reply
Why is C wrong?
Hello, wouldn’t answer choice C allow us to make the same deduction?
Daniellelagos on January 15 at 03:33PM
  • December 2010 LSAT
  • SEC4
  • Q6
1
Reply
'S' in 5th
Why is this even a question? am i missing something? S has to come before T and U? why will S app...
Bukola on January 8, 2023
  • December 2010 LSAT
  • SEC4
  • Q21
1
Reply
R with O?
Am I missing something? Why can't O be with R in the last question?
MayaM on August 21, 2022
  • December 2010 LSAT
  • SEC4
  • Q13
1
Reply
c & E
Hi that question is tricking because the places are not lined up correctly. C looks like it has 3...
Rhena on January 19, 2022
  • December 2010 LSAT
  • SEC4
  • Q14
2
Replies
Approaching this problem
How is it you can be comfortable with listing out what can and cannot be options for R and checki...
Meredith on June 4, 2021
  • December 2010 LSAT
  • SEC4
  • Q16
3
Replies
Contrapostive of last rule
Still confused on how the contrapostive of this makes sense? "If a window does not contain purpl...
DanielM on January 7, 2021
  • December 2010 LSAT
  • SEC4
  • Q7
1
Reply
2-2-2-2-0 & 2-2-2-2-1 ??
Hi, Could you please explain how to come up with the numerical deduction 2-2-2-2-0 and 2-2-2-2...
Rozanna on January 4, 2021
  • December 2010 LSAT
  • SEC4
  • Q15
1
Reply
Why doesn't answer choice D work? I can't seem ...
I placed Y,P, and R within window 1, G,P,O, and R within window 2, and O and G in window 3
ielkind on August 5, 2020
  • December 2010 LSAT
  • SEC4
  • Q11
6
Replies
purple and Orange
in Rule 4 it says if a glass does not contain purple glass then it contains orange glass. doesn't...
shannonk68 on August 4, 2020
  • December 2010 LSAT
  • SEC4
  • Q10
1
Reply
Hypothetical
As you can see, this game tripped me out a but :P. In the hypothetical drawn in the video, the...
SorooshKosha on July 23, 2020
  • December 2010 LSAT
  • SEC4
  • Q15
4
Replies
Orange & Purple
I'm confused about the orange and purple. I thought they couldn't go together and I still don't u...
brianbasuperez on July 11, 2020
  • December 2010 LSAT
  • SEC4
  • Q8
1
Reply
Question 20
If T in in spot 1 TM block How can 6 be M ? TM or WM rule 3 T M S U R/W...
scottdowns on July 8, 2020
  • December 2010 LSAT
  • SEC4
  • Q20
3
Replies
Why can’t T be in spot 5?
The only Not Law in 5 is P so why can’t G/s/T be in 5? Meaning T wouldn’t have to be in 1
Reina on June 17, 2020
  • December 2010 LSAT
  • SEC4
  • Q2
1
Reply
Question
I listened the video but still don't understand why the other answers are wrong. It seems as thou...
meisen on June 16, 2020
  • December 2010 LSAT
  • SEC4
  • Q6
2
Replies
Game Setup
When I originally attempted this game I made the deduction that G & S will always be in spaces 4,...
LaurenL on June 2, 2020
  • December 2010 LSAT
  • SEC4
  • Q1
1
Reply
The last two constraints
Hey there, I got confused on the last two constraints, in which the video explained that Q & T...
SorooshKosha on March 11, 2020
  • December 2010 LSAT
  • SEC4
  • Q14
2
Replies
Question
I noticed while I was trying to do this game there is hardly any room to diagram the information....
SROTD702 on January 9, 2020
  • December 2010 LSAT
  • SEC4
  • Q1
1
Reply